Difference between revisions of "2014 AMC 12B Problems/Problem 20"

(Added templates)
 
(6 intermediate revisions by 4 users not shown)
Line 6: Line 6:
 
\textbf{(C) }19\qquad
 
\textbf{(C) }19\qquad
 
\textbf{(D) }20\qquad
 
\textbf{(D) }20\qquad
\textbf{(E) }</math> infinitely many<math>\qquad</math>
+
\textbf{(E) }\text{infinitely many}\qquad</math>
  
 
==Solution==
 
==Solution==
 
The domain of the LHS implies that <cmath>40<x<60</cmath> Begin from the left hand side
 
The domain of the LHS implies that <cmath>40<x<60</cmath> Begin from the left hand side
 
<cmath>\log_{10}[(x-40)(60-x)]<2</cmath>
 
<cmath>\log_{10}[(x-40)(60-x)]<2</cmath>
 +
<cmath>(x-40)(60-x)<100</cmath>
 
<cmath>-x^2+100x-2500<0</cmath>
 
<cmath>-x^2+100x-2500<0</cmath>
 
<cmath>(x-50)^2>0</cmath>
 
<cmath>(x-50)^2>0</cmath>
Line 16: Line 17:
 
Hence, we have integers from 41 to 49 and 51 to 59. There are <math>\boxed{\textbf{(B)} 18}</math> integers.
 
Hence, we have integers from 41 to 49 and 51 to 59. There are <math>\boxed{\textbf{(B)} 18}</math> integers.
  
 +
==Solution 2==
 +
This solution is similar to the first solution, but perhaps more clear.
 +
 +
By the properties of logarithms, <math>\log_{10}(x-40)+\log_{10}(60-x)=\log_{10}((x-40)(60-x))\implies\log_{10}((x-40)(60-x)).</math> Therefore, <math>(x-40)(60-x)<100.</math> We can see that this function is concave down (that is, it looks like an upside-down U shape); that is, it has a maximum value. This maximum value is attained at <math>x=50</math>, in which <math>(x-40)(60-x)=100.</math> This is the one point on the function that does not satisfy the given condition.
 +
 +
We also know that, as noted in solution 1, <math>x</math> must be greater than <math>40</math> and less than <math>60</math>, since otherwise, <math>\log_{10}(x-40)</math> or <math>\log_{10}(60-x)</math> would be undefined. Thus, the possible values of <math>x</math> are the positive integers from <math>41</math> to <math>59,</math> inclusive, excluding <math>50.</math> Thus, there are <math>19-1=18</math> total integers.
 +
 +
~ Technodoggo
 +
 +
== Video Solution by OmegaLearn ==
 +
https://youtu.be/RdIIEhsbZKw?t=1088
 +
 +
~ pi_is_3.14
 +
 +
== See also ==
 
{{AMC12 box|year=2014|ab=B|num-b=19|num-a=21}}
 
{{AMC12 box|year=2014|ab=B|num-b=19|num-a=21}}
 
{{MAA Notice}}
 
{{MAA Notice}}

Latest revision as of 22:03, 14 September 2023

Problem

For how many positive integers $x$ is $\log_{10}(x-40) + \log_{10}(60-x) < 2$ ?

$\textbf{(A) }10\qquad \textbf{(B) }18\qquad \textbf{(C) }19\qquad \textbf{(D) }20\qquad \textbf{(E) }\text{infinitely many}\qquad$

Solution

The domain of the LHS implies that \[40<x<60\] Begin from the left hand side \[\log_{10}[(x-40)(60-x)]<2\] \[(x-40)(60-x)<100\] \[-x^2+100x-2500<0\] \[(x-50)^2>0\] \[x \not = 50\] Hence, we have integers from 41 to 49 and 51 to 59. There are $\boxed{\textbf{(B)} 18}$ integers.

Solution 2

This solution is similar to the first solution, but perhaps more clear.

By the properties of logarithms, $\log_{10}(x-40)+\log_{10}(60-x)=\log_{10}((x-40)(60-x))\implies\log_{10}((x-40)(60-x)).$ Therefore, $(x-40)(60-x)<100.$ We can see that this function is concave down (that is, it looks like an upside-down U shape); that is, it has a maximum value. This maximum value is attained at $x=50$, in which $(x-40)(60-x)=100.$ This is the one point on the function that does not satisfy the given condition.

We also know that, as noted in solution 1, $x$ must be greater than $40$ and less than $60$, since otherwise, $\log_{10}(x-40)$ or $\log_{10}(60-x)$ would be undefined. Thus, the possible values of $x$ are the positive integers from $41$ to $59,$ inclusive, excluding $50.$ Thus, there are $19-1=18$ total integers.

~ Technodoggo

Video Solution by OmegaLearn

https://youtu.be/RdIIEhsbZKw?t=1088

~ pi_is_3.14

See also

2014 AMC 12B (ProblemsAnswer KeyResources)
Preceded by
Problem 19
Followed by
Problem 21
1 2 3 4 5 6 7 8 9 10 11 12 13 14 15 16 17 18 19 20 21 22 23 24 25
All AMC 12 Problems and Solutions

The problems on this page are copyrighted by the Mathematical Association of America's American Mathematics Competitions. AMC logo.png